LSAT and Law School Admissions Forum

Get expert LSAT preparation and law school admissions advice from PowerScore Test Preparation.

 Administrator
PowerScore Staff
  • PowerScore Staff
  • Posts: 8917
  • Joined: Feb 02, 2011
|
#36443
Complete Question Explanation
(See the complete passage discussion here: lsat/viewtopic.php?t=7465)

The correct answer choice is (E)

Since the general structure of the passage is one of introduction, followed by mention of criticism,
followed by defense, you must select a response that captures that overall fl ow.

Answer choice (A): Even though it might be acceptable to refer to the Hippocratic oath as a
“principle,” the author does not clearly modify the oath. The author suggests how the periphery of
the oath might change, and even gives an example; however, at the core of that change is the entirely
unmodifi ed principle of benefi cence.

In short, the best reason for rejecting this somewhat attractive choice is that the author believes that
the central principle of the oath must remain intact, but this choice claims that the author modifi es a
principle.

Answer choice (B): This choice fails to mention anything that could refer to the fi rst 12 lines of the
passage. Furthermore, this choice implies that the author accepts, or fails to counter, the criticisms in
lines 13-33, but that is false. The entire second paragraph is devised to reject many of the criticisms
in lines 13-33, so this response is incorrect. If you chose this response, you probably read it too
quickly and thought that it stated that the author rejects criticisms of the oath; however, that is a
misread because the answer states that “replies to those criticisms are…dismissed,” not the criticisms
themselves.

Answer choice (C): The author only discusses a small part of the history of the Hippocratic oath,
so this response is immediately weak. Furthermore, the author largely rejects criticisms of the oath,
and you should not assume that since the author fails to address certain critiques that he then accepts
them. In fact, by clearly asserting the core value of benefi cence (the individual patient’s benefi t), the
author indirectly discards the notion that market forces should be given full reign and that modern
issues should signifi cantly alter the main principle of the oath. Lastly, the author discussed how the
periphery of the code might be modifi ed, but did not directly modify or construct a new code.

Answer choice (D): Since the author did not formulate the Hippocratic oath, but instead merely
reported what the oath is, the passage does not justify this response. Furthermore, this response
incorrectly places the defense of the oath out of the correct order in the passage.

Answer choice (E): This is the correct answer choice. This response mirrors the map of the
passage that you could have prephrased. First, the author introduces the oath. Second, some
criticisms are described. Third, the oath is defended. Furthermore, you will note that this response
more correctly captures the fact that the author included more than a principle but less than a history
in the fi rst 12 lines (“The tradition”), and more correctly captures the general tone of the author’s
defense in the last paragraph in that the author did not reject the idea of modifying the periphery of
the oath, but did believe that on the whole the oath should be retained.
 mpoulson
  • Posts: 148
  • Joined: Mar 25, 2016
|
#28510
Hello,

I found the difference between D and E to be very slight and mistakenly chose D. I guess I misunderstood what the author was saying. I thought when the author suggested areas of the oath that were being revised and reinterpreted it made sense that he was only partially supporting the oath overall. Can you explain how I am off track and can you explain what makes D wrong?
 David Boyle
PowerScore Staff
  • PowerScore Staff
  • Posts: 836
  • Joined: Jun 07, 2013
|
#28591
mpoulson wrote:Hello,

I found the difference between D and E to be very slight and mistakenly chose D. I guess I misunderstood what the author was saying. I thought when the author suggested areas of the oath that were being revised and reinterpreted it made sense that he was only partially supporting the oath overall. Can you explain how I am off track and can you explain what makes D wrong?

Hello mpoulson,

D is not a terrible answer, but the author's discussion of the oath is a little too loose to be called a "general principle". But clearly, the author likes the oath and is defending it, so, answer E. (Which says "general defense", i.e., he/she is defending the oath in general, even if admitting that it could use some revision)

Hope this helps,
David

Get the most out of your LSAT Prep Plus subscription.

Analyze and track your performance with our Testing and Analytics Package.